How do I correctly prove the memoryless property of a geometric distribution?

  • Thread starter Thread starter cookiesyum
  • Start date Start date
  • Tags Tags
    Property
Click For Summary
SUMMARY

The discussion focuses on proving the memoryless property of a geometric distribution, specifically the equation P(X ≥ k + j | X ≥ k) = P(X ≥ j) for nonnegative integers k and j. The user initially attempts to derive this property but encounters algebraic errors in their proof. The correct approach involves recognizing that P(X ≥ k + j) can be simplified to (1 - p)^(k + j) and P(X ≥ k) to (1 - p)^k, leading to the conclusion that the ratio simplifies to (1 - p)^j, confirming the memoryless property.

PREREQUISITES
  • Understanding of geometric distribution and its properties
  • Knowledge of conditional probability
  • Familiarity with algebraic manipulation of probabilities
  • Basic concepts of random variables
NEXT STEPS
  • Study the derivation of the memoryless property in geometric distributions
  • Explore conditional probability in depth
  • Learn about other memoryless distributions, such as the exponential distribution
  • Practice algebraic proofs involving probability functions
USEFUL FOR

Students studying probability theory, statisticians, and anyone interested in understanding the properties of geometric distributions and their applications in statistical modeling.

cookiesyum
Messages
72
Reaction score
0

Homework Statement



Let X have a geometric distribution. Show that

P(x>or= k+j|x>or=k) = P(X>or=j)

where k and j are nonnegative integers.

The Attempt at a Solution



P(x>or= k+j|x>or=k) = P(x>or= k+j intersect x>or=k)/P(x>or=k) = P(x>or=k+j)/P(x>or=k) for j>or= 0 = [1-P(x=k+j)]/[1-P(x=k)] = [1 - p(1-p)^(k+j)]/[1-p(1-p)^k] = [1-p(1-p)^k(1-p)^j]/[1-p(1-p)^k] = (1-p)^j which is equal to P(x>or=j) if it had another p so that it was p(1-p)^j...so where am I losing this p in my proof?
 
Physics news on Phys.org
= [1-P(x=k+j)]/[1-P(x=k)]

That's an error. It should be = [1-P(x<k+j)]/[1-P(x<k)]. However, don't do it this way.

= [1-p(1-p)^k(1-p)^j]/[1-p(1-p)^k] = (1-p)^j

That's an algebra error. Are you saying (a+bcd)/(a+bc) would be equal to d? It isn't.

(1-p)^j which is equal to P(x>or=j)

This is right, and it is the key. Since you know this, you should go back and use this same idea in the expression P(x>or=k+j)/P(x>or=k).
 
Question: A clock's minute hand has length 4 and its hour hand has length 3. What is the distance between the tips at the moment when it is increasing most rapidly?(Putnam Exam Question) Answer: Making assumption that both the hands moves at constant angular velocities, the answer is ## \sqrt{7} .## But don't you think this assumption is somewhat doubtful and wrong?

Similar threads

Replies
4
Views
1K
  • · Replies 37 ·
2
Replies
37
Views
7K
Replies
3
Views
1K
  • · Replies 8 ·
Replies
8
Views
1K
  • · Replies 3 ·
Replies
3
Views
2K
  • · Replies 3 ·
Replies
3
Views
1K
  • · Replies 15 ·
Replies
15
Views
6K
Replies
2
Views
1K
  • · Replies 1 ·
Replies
1
Views
2K
  • · Replies 16 ·
Replies
16
Views
6K